I don't understand this question can someone help me with this please, please explain!! and thank you for the help!! If u dont know try your best :D

I Don't Understand This Question Can Someone Help Me With This Please, Please Explain!! And Thank You

Answers

Answer 1

Answer:

a. Find the largest and smallest numbers and then subtract the smallest from the largest number.

b. 52

Step-by-step explanation:

Range Definition: Difference between largest and smallest numbers.

1. Find the largest and smallest numbers;

Biggest: 17|5 (175)

Smallest: 12|3 (123)

2. Subtract the smallest number from the largest number;

175 - 123 = ?

? = 52


Related Questions

please help me
IF YOU DONT KNOW THE QUESTION, PLEASE DONT ANSWER IT FOR THE PONIS. I REALLY NEED IT

Answers

Answer:

As follows,

Step-by-step explanation:

1

3x^2y^-1

3x^2/y

2

∛2x^5y^3

2^(1/3)x^(5/3)y^(3/3)

2^(1/3)x^(5/3)y

3

5^(4/4)x^(1/4)y^(3/4)

∜5^4xy^3

help plz ill give brainliest

Answers

Answer:

30 gallons

Step-by-step explanation:

find the 6 on the x axis, and it meets to the 30 on the y axis, therefore the answer is 30 gallons (sorry if this doesn't make sense or if it's incorrect)

Write an equation for the line passing through
the points (4,-3) (-2, -6)

Answers

Answer:

y=1/2x-5

Step-by-step explanation:

By solving for slope, we get 1/2

We use the equation b=y-mx to find the y intercept

b= -5

Equation: y= 1/2x - 5


58
4 25
What is the mixed number of decimals

Answers

Answer:

fun fact : by reading what i just right u just lost 5 sec of your life

Step-by-step explanation:

K(–5, –9); rotation 180° about the origin, translation 3 units left

Answers

Answer:

Step-by-step explanation:

Rotate 180 degrees about the origin

X(x,y) ====> X'(-x,-y)

K(-5 , - 9) = K ' (- -5, - - 9)

K' (5,9)

3 Units Left.

y is unchanged.

x becomes x - 3

K'(5,9) ===>k" (5 - 3,9)

k"(2,9)

The leftward motion does not look done the same way as moving a parabola left. But you are just changing the x value to move a point left. You are not imposing other rules on the figure.

PLEASE ANSWER SO I DON’T FAIL THIS


Copy Z DEF to the line so that S is the

vertex.

This task will be complete when you have

constructed an angle with vertex S that is

congruent to ZDEF..

Answers

So the 6 plus the other active numbers are going to be multiplied by the nearest ten to get the answer which is 17

A snack food company decreases the size of their single-serving size tortilla chips bag to 3.0
3.
0
ounces, which is 10%
10
%
less than the original size.

What is the original size of the bag?
Round your answer to the nearest tenth.

Answers

Maybe 10% because if u were above it it would have been

Cat food costs $2.85 for five cans. Ben only wants to buy one can. How much will it cost?

Answers

Answer:

$0.57 for each can of cat food

Step-by-step explanation:

2.85/5 = 0.57 giving you the answer

Answer:

it would cost 0.57 cents!

Explanation:

you'd have to divide 2.85 by 5, because if it is that much for 5 cans, and he only wants to buy 1, you'd divide it by 5, to get 0.57 :)

NEED HELP! John wrote the numbers 678,901 and 67,890. How many times greater is the value of 7 in 678,901 than the value of 7 in 67,890?A. 10,000B. 1,000C. 100D. 10

Answers

the answer is D. 10

70,000 / 7,000 = 10

Find the value of x.

Answers

Answer:

for this problem we use the sine function

[tex] \sin(x) = \frac{opp}{hyp} \\ \sin(38 ) = \frac{x}{100} [/tex]

multiply both sides of the equation by 100

[tex]100 \sin(38) = \frac{100x}{100} [/tex]

100 cancels on the right hand side of the equation

[tex]100 \sin(38) = x \\ x = 61.566\:ft[/tex]

Can you help me please​

Answers

Answer:

a. 8, 14, 20

x=5 when y=11

Step-by-step explanation:

Just subsititute the x in the equation with the number that represents x.

Part E
? Question
How would the situation change if the interest on Emma's credit card were compounded annually
rather than monthly, and she didn't make any payments toward the balance?
Select the correct answer from each drop-down menu.
After 4 years, Emma would owe approximately $ M for her original purchase of $300.
It would take around years for her balance to increase from $300 to $450.

Answers

Answer:

$525

2.9 years

Step-by-step explanation:

This question is about financial management. It is to be noted that it would take around 2.9  years for her balance to increase from $300 to $450.

What is the calculation for the above?

Note that according to the question,

After 4 years, Emma would owe about $525 for her original purchase of $300.

The equation will be:

300 * (1 + x)[tex]^{4}[/tex] = 525 (This is based on the compound interest formula_

⇒ (1+x)[tex]^{4}[/tex] = 7/4 (If we divide both sides by 300)

Hence

x = 0.1502

= 15.02%

When the balance ups from $300 to $450,

We can state that:

300 * ( 1+ 15.02%) [tex]^{y}[/tex] = 450

⇒ Hence, y = 2.9 years.

Learn more about financial management at:
https://brainly.com/question/17155837
#SPJ6

B: Josh starts off the weekend with $5,25 and earns $7.50 for each hour that he works. At
weekend he has $57,76. How many hours did he work?


PLEASE HELP I WILL GIVE BRAINLY THINGY

Answers

Answer:

23hrs

Step-by-step explanation:

Can someone explain how to write a fraction in simplest form? Example: 5/8-(7/8) I’ll mark as brainliest!!!!

Answers

Answer:

-2/8 or 1/4

Step-by-step explanation:

5/8-7/8, u sbtract the numerator then divide

subtract the numetrator=5-7=-2

divide= -2/8=-1/4


Mike went to the store to buy shirts. The shirts were $45 each. The store was offering the following discount:
Buy 1 shirt at regular
price and get any 2
shirts of the same
price for 40% off!
Mike decided to buy 3 shirts. Not including sales tax, how much money did Mike save by buying the shirts at the discounted price?
$36
$54
$81
$99

Answers

Answer:

Mike saved 54 dollars

Step-by-step explanation:

because 40% of $45 is $18. so that makes the other 2 shirts 27 dollars a piece. so he saves  a lot of money.

You need 520 mL of a 15% alcohol solution. On hand, you have a 65% alcohol mixture. How much of the 65% alcohol mixture and pure water will you need to obtain the desired solution?

You will need _____ mL of pure water and _____ mL of the 65% solution.

Answers

Answer:

400 ml, 120 ml

Step-by-step explanation:

Alcohol content of 15% solution:

0.15*520 = 78 ml

Volume of 65% solution containing 78 ml of alcohol:

78/0.65 = 120 ml

Required water volume to obtain 15% solution:

520 - 120 = 400 ml

3(6d + 1) = ?
How do I see what the d = to multiply it 3x6d but it doesn’t shows what the d= does the d = 1 since it’s only 1 d? Or how do I simplify this

Answers

Answer:

18d+3

Step-by-step explanation:

start with 3(6d+1)

separate the problem into 3(6d)+3(1)

simplify to get your answer:

18d+3

Answer:

18d + 3

Step-by-step explanation:

3 ( 6d + 1 )

= ( 3 x 6d ) + ( 3 x 1 )

= 18d + 3

Plz help it’s due tonight!!!

Answers

Answer: AB≅EF, ∠B≅∠E, BC≅DE

Step-by-step explanation:

AB and EF are both marked with one line, making them congruent.

The same goes for BC and DE since they are marked with two lines.

Angles B and E are congruent because of the marking that is specified in both figures.

what is If 1/2 a pound of walnuts cost $3.25. home much is one pound of walnuts?

Answers

Answer:

6.50

Step-by-step explanation:

just do 3.25 times 2 hope it helps

Your answer would be 6.50

Please help with #4!!

Answers

Answer:

I think if you want to enlarge it add 10 to all three sides to get the sides of 13 16 and 16

What is the value of x in the triangle below?
ModuleTwoPostAs...
a
>> Q
3x
T
V
8x - 30
R
15
45
6
18

Answers

Answer:

d

Step-by-step explanation:

−5(10) Helpppp please

Answers

Step-by-step explanation:

I will help you if you follow me

Answer:

-50

Step-by-step explanation:

all you do is -5 times 10

Which of the following rational number between 3/7 and 7/8 ?
13/56
19/56
27/56
53/56

Answers

Answer:

Step-by-step explanation:

There are two ways to do this. You could use your calculator to find out the decimal numbers that are equivalent to the two given fractions. Then check which one of the givens falls between these two numbers. I wouldn't do it that way -- not when you are told that the answer has 56 as the denominator.  So ...

The best way to do it is to change the two givens so that they have a denominator of 56

3/7 = 3*8/7*8 = 24 / 56

7*7 / 7*8 = 49 / 46

So you need a number that is between 49 and 24

There is only one 27 / 56

1. What is the constant of proportionality?

2. What is the equation?

Answers

Answer:

y = 1/3a

Step-by-step explanation:

y = ka

1 = 3k

k = 1/3

Confirm constant: (4-1)/(12-3) = 3/9 = 1/3

1 = 3(1/3) + b

1 = 1 + b

b = 0

y = 1/3x or 1/3a

Which best describes the graph of the function f(x) = 4(1.5)x?

Answers

In the question "Which best describes the graph of the function f(x) = 4(1.5)^x?" The correct answer is "The graph passes through the point (0, 4), and for each increase of 1 in the x-values, the y-values increase by a factor of 1.5." (option B) Because when x = 0, f(0) = 4(1.5)^0 = 4(1) = 4. Thus the graph passes through (0, 4). Also for every increase of 1 in the x-values, the y-values increase by a factor of 1.5 i.e. f(0) = 4(1.5)^0 f(1) = 4(1.5)^1 f(x) = 4(1.5)^2 etc

Read more at Answer.Ya.Guru – https://answer.ya.guru/questions/3695268-which-best-describes-the-graph-of-he-function-f-x-4-15-x.html#answer4949036

Answer:

B

Step-by-step explanation:

Since this Equation isn't linear because of the squaring, we can eliminate A and C which imply linear functions. Then it's as easy as pluging in zero for the equation and seeing which one works, and you get an answer of B

(sinα+cosα)^2−(sinα−cosα)^2​

Answers

Answer:

Error

Step-by-step explanation:

The answer is Error u might want to ask ur teacher

Solve for x ... please help

Answers

Answer:

[tex]7x + 49 \degree = 2x + 94\degree \\ 7x - 2x = 94\degree - 49 \degree \\ 5x = 45\degree \\ x = \frac{45}{5} \\ x = 9 \\ check \\ 7(9) + 49 = 112\degree \\ 2(9) + 94 = 112\degree[/tex]

Determine if the sequence 0.2 comma 1 comma 5 comma 25 comma... is arithmetic or geometric. Then identify the next term in the sequence.

A
geometric; 125

B
arithmetic; 75

C
arithmetic; 125

D
geometric; 75

Answers

Answer:

The answer would be C.

Step-by-step explanation:

0.2 x 5 = 1

1 x 5 =5

5 x 5 = 25

So your next number in the sequence should be 125.

Could someone please help answer this thanks so much, please no spam, will give brainliest (homework help) thank you!

Answers

Answer:

x > -7

make sure to put the sign with the line under it because I can't type it

3) if the cost of 1 pen is Rs.10, find the cost of 5 pen​

Answers

Answer: 5 pens cost Rs.50

Explanation: Since 1 pen costs Rs.10, you will need to multiply the number of pens (1) and the cost of that number of pens (Rs.10) by the amount of pens you must find the cost for (5). Since 10 times 5 is 50, that means that 50 is the cost of 5 pens.

I hope this helps!

Other Questions
Why tides pass over all world? Other effects of this phenomenon predicate adjective are in the predicate and follow a verb of being or a linking verb like seem,look,feels.write the predicate adjective that makes Sense given the information Help help help please Easy points. 1. how can your teachers help you become a better student?2. How can parents help you become a better student? Which of the following best describes the purpose of child labor laws?to protect children from hazardous work environmentsto prevent children from entering the workforceto create new opportunities for children to workto ensure that all children attend school\ all students = 700450 = choose to enrolled on an off-grid class320 = on an online class275 = on both off-grid and online class1) How many parents choose only off-grid class for their children? 2) Is there any parents choose not to enroll their children in this school year? If yes, how many? 3) How many parents choose an online class for their children only? 4) How many parents do not choose online class? 5) How many parents do not use off-grid class?help pls, I rlly need it now. Am I correct? I WILL GIVE BRAINILEST TO PERSON THT HELPS ME FIRST!Ty if u do UWU Is (2,3), a solution to the system of equation x2 + y2 = 13 and 2x - y=4?TrueFalse What made the civil right act of 1886 significant? PLEASE HELP!!!! WILL GIVE BRAINLIEST!!!!!What is the overall structure of this article? Which number can each term of the equation be multiplied by to eliminate the fractions before solving? -3/4m-1/2=2+1/4m A solar eclipse occurs during the day and can occur only during what phase of the moon?New moonFull moon How have people adapted to meet the challenges of the anctarctica circumstances Based on the fourth amendment if a person was pulled over driving a motor home that they lived in would the government have to treat it like a house or like vehicle? Consider two future observatories in space. Observatory X consists of a single 50-meter telescope. Observatory Y is an interferometer consisting of five 10-meter telescopes, spread out over a region 100 meters across. Which observatory can detect dimmer stars, and which one can see more detail in its images Helps nervous system control and coordinate activities of the body; helps regulate growth.What system is that help me please what are my motivations,my goals, my values are all fungi eukariotic? help,help,help, help ,help ,helppppppppppppppppppppppppppppppppppppppppppppppppppppppppppppppppppppppppppppppppppppppppppppppppppppppppppppppppppppppppppppppppppppppppppppppppppppppppppppppppppppppppppppppppppppppppppppppppppppppppppppppppppppppppppppppppppppppppppppppppppppppppppppppppppppppppppppppppp There are a total of 84 campers attending a summer camp. The ratio of boys to girls is 4 to 3.How many girls are attending the summer camp?Enter your answer in the box.